Zbieżność szeregu

Granice, pochodne, całki, szeregi
Otrzymałeś(aś) rozwiązanie do zamieszczonego zadania? - podziękuj autorowi rozwiązania! Kliknij
lukash-17
Rozkręcam się
Rozkręcam się
Posty: 31
Rejestracja: 29 gru 2017, 20:35
Podziękowania: 21 razy
Płeć:

Zbieżność szeregu

Post autor: lukash-17 »

Pomoże ktoś rozwiązać te zadania?

Zbadać zbieżność szeregów:
Obrazek
radagast
Guru
Guru
Posty: 17549
Rejestracja: 09 lis 2010, 07:38
Lokalizacja: Warszawa
Podziękowania: 41 razy
Otrzymane podziękowania: 7435 razy
Płeć:

Post autor: radagast »

trzeba przepisać , a nie wstawiać obrazki !
Pierwszy rozbieżny, tylko też nie chce mi się pisać...
lukash-17
Rozkręcam się
Rozkręcam się
Posty: 31
Rejestracja: 29 gru 2017, 20:35
Podziękowania: 21 razy
Płeć:

Post autor: lukash-17 »

\sum_{}^{}
lukash-17
Rozkręcam się
Rozkręcam się
Posty: 31
Rejestracja: 29 gru 2017, 20:35
Podziękowania: 21 razy
Płeć:

Post autor: lukash-17 »

Właśnie chciałem tak zrobic ale mam problem żeby to poprawnie przepisać :(
radagast
Guru
Guru
Posty: 17549
Rejestracja: 09 lis 2010, 07:38
Lokalizacja: Warszawa
Podziękowania: 41 razy
Otrzymane podziękowania: 7435 razy
Płeć:

Post autor: radagast »

no to ja też :(
lukash-17
Rozkręcam się
Rozkręcam się
Posty: 31
Rejestracja: 29 gru 2017, 20:35
Podziękowania: 21 razy
Płeć:

Post autor: lukash-17 »

a) \(\sum_{n=1}^{ \infty } \left( \frac{2n^3+4n2-1}{2n^3+3n} \right)^{3n^2+2}\)
radagast
Guru
Guru
Posty: 17549
Rejestracja: 09 lis 2010, 07:38
Lokalizacja: Warszawa
Podziękowania: 41 razy
Otrzymane podziękowania: 7435 razy
Płeć:

Post autor: radagast »

\(\displaystyle \sum_{n=1}^{ \infty } \left( \frac{2n^3+4n^2-1}{2n^3+3n} \right)^{3n^2+2}\)
z kryterium Cauchy'ego:
\(\sqrt[n]{\left( \frac{2n^3+4n^2-1}{2n^3+3n} \right)^{3n^2+2}} =\left( \frac{2n^3+4n^2-1}{2n^3+3n} \right)^{ \frac{3n^2+2}{n} } =\left( \frac{2n^3+3n+4n^2-3n-1}{2n^3+3n} \right)^{ \frac{3n^2+2}{n} } =\left(1+ \frac{4n^2-3n-1}{2n^3+3n} \right)^{ \frac{3n^2+2}{n} } =\\
\left(1+ \frac{4n^2-3n-1}{2n^3+3n} \right)^{ \frac{2n^3+3n}{4n^2-3n-1} \cdot \frac{4n^2-3n-1}{2n^3+3n} \cdot \frac{3n^2+2}{n} } =e^{\frac{4n^2-3n-1}{2n^3+3n} \cdot \frac{3n^2+2}{n}}=e^6>1\)

-rozbieżny
radagast
Guru
Guru
Posty: 17549
Rejestracja: 09 lis 2010, 07:38
Lokalizacja: Warszawa
Podziękowania: 41 razy
Otrzymane podziękowania: 7435 razy
Płeć:

Post autor: radagast »

drugi też rozbieżny.
lukash-17
Rozkręcam się
Rozkręcam się
Posty: 31
Rejestracja: 29 gru 2017, 20:35
Podziękowania: 21 razy
Płeć:

Post autor: lukash-17 »

b) \(\sum_{n=1}^{ \infty } \left( \sqrt[3]{n^3+2n^2} -n \right)\)
c) \(\sum_{n=1}^{ \infty } \frac{3^nn!}{ \left(2n \right)^n }\)
d) \(\sum_{n=1}^{ \infty } \left(-1 \right)^n \frac{n^3}{2^{2n}+3^n}\)
e) \(\sum_{n=1}^{ \infty } \left(-1 \right)^n \frac{n}{n^2+2}\)
radagast
Guru
Guru
Posty: 17549
Rejestracja: 09 lis 2010, 07:38
Lokalizacja: Warszawa
Podziękowania: 41 razy
Otrzymane podziękowania: 7435 razy
Płeć:

Post autor: radagast »

\(\displaystyle \sum_{n=1}^{ \infty } \left( \sqrt[3]{n^3+2n^2} -n \right)\)
\(\Lim_{n\to \infty } \left( \sqrt[3]{n^3+2n^2} -n \right)= \frac{2}{3} \neq 0\) - szereg nie spełnia warunku koniecznego zbieżności


\(\Lim_{n\to \infty } \left( \sqrt[3]{n^3+2n^2} -n \right) \cdot \frac{ \sqrt[3]{n^3+2n^2}^2+ \sqrt[3]{n^3+2n^2} \cdot n+n^2}{ \sqrt[3]{n^3+2n^2}^2+ \sqrt[3]{n^3+2n^2} \cdot n+n^2}=\Lim_{n\to \infty } \frac{n^3+2n^2 -n^3}{ \sqrt[3]{n^3+2n^2}^2+ \sqrt[3]{n^3+2n^2} \cdot n+n^2}= \Lim_{n\to \infty } \frac{2n^2 }{ \sqrt[3]{n^3+2n^2}^2+ \sqrt[3]{n^3+2n^2} \cdot n+n^2}=\\
\Lim_{n\to \infty } \frac{2 }{ \sqrt[3]{1+2 \frac{1}{n} }^2+ \sqrt[3]{ 1 + \frac{2}{n} } +1}= \frac{2}{3}\)
radagast
Guru
Guru
Posty: 17549
Rejestracja: 09 lis 2010, 07:38
Lokalizacja: Warszawa
Podziękowania: 41 razy
Otrzymane podziękowania: 7435 razy
Płeć:

Post autor: radagast »

trzeci wreszcie jest zbieżny (kryterium d'Alemberta)
lukash-17
Rozkręcam się
Rozkręcam się
Posty: 31
Rejestracja: 29 gru 2017, 20:35
Podziękowania: 21 razy
Płeć:

Post autor: lukash-17 »

Czy ten zapis:
\(\sqrt[3]{n^3+2n^2}^2\)
oznacza następujący?
\(\left( \sqrt[3]{n^3+2n^2}\right)^2\)
lukash-17
Rozkręcam się
Rozkręcam się
Posty: 31
Rejestracja: 29 gru 2017, 20:35
Podziękowania: 21 razy
Płeć:

Post autor: lukash-17 »

Czy to co zrobiłem jest dobrze?
Jeśli tak to co dalej?

c) \(\sum_{n=1}^{ \infty } \frac{3^nn!}{ \left(2n \right)^n }\)

z kryterium d'Alemberta:

\(\Lim_{x\to \infty } \frac{ \frac{3^{n+1}(n+1)!}{(2n+1)^{n+1}} }{\frac{3^nn!}{ \left(2n \right)^n }}

= \Lim_{x\to \infty } \frac{3^{n+1}(n+1)!}{(2n+1)^{n+1}}* \frac{ \left(2n \right)^n }{3^nn!}

= \Lim_{x\to \infty } \frac{ \left(2n \right)^n }{ \left(2n+1 \right) \left(2n \right)^n }* \frac{3^{n+1} \left(n+1 \right)! }{3^nn!}

= \Lim_{x\to \infty } \frac{1}{ \left(2n+1 \right) } * \frac{3^{n+1} \left(n+1 \right)! }{3^nn!}\)
radagast
Guru
Guru
Posty: 17549
Rejestracja: 09 lis 2010, 07:38
Lokalizacja: Warszawa
Podziękowania: 41 razy
Otrzymane podziękowania: 7435 razy
Płeć:

Re:

Post autor: radagast »

lukash-17 pisze:Czy ten zapis:
\(\sqrt[3]{n^3+2n^2}^2\)
oznacza następujący?
\(\left( \sqrt[3]{n^3+2n^2}\right)^2\)
Tak.
radagast
Guru
Guru
Posty: 17549
Rejestracja: 09 lis 2010, 07:38
Lokalizacja: Warszawa
Podziękowania: 41 razy
Otrzymane podziękowania: 7435 razy
Płeć:

Re:

Post autor: radagast »

lukash-17 pisze:
\(\Lim_{x\to \infty } \frac{ \frac{3^{n+1}(n+1)!}{(2n+1)^{n+1}} }{\frac{3^nn!}{ \left(2n \right)^n }}

= \Lim_{x\to \infty } \frac{3^{n+1}(n+1)!}{(2n+1)^{n+1}}* \frac{ \left(2n \right)^n }{3^nn!}

= \Lim_{x\to \infty } \frac{ \left(2n \right)^n }{ \left(2n+1 \right) \left(2n \right)^n }* \frac{3^{n+1} \left(n+1 \right)! }{3^nn!}

= \Lim_{x\to \infty } \frac{1}{ \left(2n+1 \right) } * \frac{3^{n+1} \left(n+1 \right)! }{3^nn!}\)
Coś nie tak...
Policzę Ci to po Nowym Roku, a może ktoś mnie ubiegnie. Wczoraj liczyłam i wychodziła mi ta granica \(\frac{3}{2} \cdot \frac{1}{e}\).
Dobrego Sylwestra i szczęśliwego nowego roku :)
ODPOWIEDZ